GMAT Exam  >  GMAT Tests  >  Practice Questions for GMAT  >  Test: Assumptions - GMAT MCQ

Test: Assumptions - GMAT MCQ


Test Description

10 Questions MCQ Test Practice Questions for GMAT - Test: Assumptions

Test: Assumptions for GMAT 2024 is part of Practice Questions for GMAT preparation. The Test: Assumptions questions and answers have been prepared according to the GMAT exam syllabus.The Test: Assumptions MCQs are made for GMAT 2024 Exam. Find important definitions, questions, notes, meanings, examples, exercises, MCQs and online tests for Test: Assumptions below.
Solutions of Test: Assumptions questions in English are available as part of our Practice Questions for GMAT for GMAT & Test: Assumptions solutions in Hindi for Practice Questions for GMAT course. Download more important topics, notes, lectures and mock test series for GMAT Exam by signing up for free. Attempt Test: Assumptions | 10 questions in 20 minutes | Mock test for GMAT preparation | Free important questions MCQ to study Practice Questions for GMAT for GMAT Exam | Download free PDF with solutions
Test: Assumptions - Question 1

The Circle Multiplex has seen tremendous growth in the number of customers since it signed an exclusive contract with a leading distributor. For the last seven months, unlike previous months, almost all the shows have been booked completely. A number of customers have even been turned away for most of the shows. Consequently, the management proposes to increase its profits by raising the ticket prices slightly above the competition’s as it believes that invariably the Multiplex will be able to fill its capacity.

Which of the following statements is an assumption made by the management of The Circle Multiplex?

Detailed Solution for Test: Assumptions - Question 1

The situation described in the question is that The Circle Multiplex has experienced significant growth in the number of customers and has been consistently booking shows at full capacity for the past seven months. The management wants to increase profits by raising ticket prices slightly above the competition's prices, believing that they will still be able to fill the capacity.

The assumption made by the management is the underlying belief that a significant number of customers of The Circle Multiplex are not extremely sensitive to ticket prices. In other words, the management assumes that customers will be willing to pay slightly higher prices and still choose The Circle Multiplex over other options.

Let's now evaluate each answer choice:

(A) The Circle Multiplex has lower ticket prices than other Multiplexes in the region.
This statement is not necessarily an assumption made by the management. It could be true or false, but it doesn't directly relate to the assumption being made about customer sensitivity to ticket prices.

(B) Factors other than ticket prices such as ambiance are very important for the customers in choosing the Multiplex.
This statement is also not directly related to the assumption being made. It suggests that ambiance is important, but it doesn't address the assumption about customer sensitivity to ticket prices.

(C) No existing customer of The Circle Multiplex is going to shift for minor economic considerations.
This statement is not directly related to the assumption either. It talks about existing customers not shifting for minor economic considerations but doesn't address the assumption about customer sensitivity to ticket prices.

(D) Not a significant number of customers of The Circle Multiplex are extremely sensitive to ticket prices.
This statement directly addresses the assumption made by the management. It suggests that the management believes that a significant number of customers are not extremely sensitive to ticket prices. If this assumption is false, then raising ticket prices may result in a decrease in customers rather than an increase in profits.

(E) Most other Multiplexes around The Circle Multiplex run most of the shows with less than half the capacity filled.
This statement is not related to the assumption being made by the management. It provides information about other multiplexes' capacity utilization but doesn't address the assumption about customer sensitivity to ticket prices.

Based on the above analysis, the correct assumption made by the management of The Circle Multiplex is (D) Not a significant number of customers of The Circle Multiplex are extremely sensitive to ticket prices.

Test: Assumptions - Question 2

Editorial: Our city's public transportation agency is facing a budget shortfall. The fastest growing part of the budget has been employee retirement benefits, which are exceptionally generous. Unless the budget shortfall is resolved, transportation service will be cut, and many transportation employees will lose their jobs. Thus, it would be in the employees' best interest for their union to accept cuts in retirement benefits.

Which of the following is an assumption the editorial's argument requires?

Detailed Solution for Test: Assumptions - Question 2

Let's analyze each answer choice and determine which one is the correct assumption for the editorial's argument.

(A) The transportation employees' union should not accept cuts in retirement benefits if doing so would not be in the employees' best interest.

  • This answer choice introduces a conditional statement that the union should not accept cuts in retirement benefits if it's not in the employees' best interest. However, the argument doesn't rely on this condition, so it is not an assumption required for the argument.

(B) The only feasible way for the agency to resolve the budget shortfall would involve cutting transportation service and eliminating jobs.

  • This answer choice introduces the idea that cutting transportation service and eliminating jobs is the only feasible solution to the budget shortfall. If this were true, it would strengthen the argument's conclusion. Therefore, this answer choice is a possible assumption.

(C) Other things being equal, it is in the transportation employees' interest to have exceptionally generous retirement benefits.

  • This answer choice introduces the idea that it is generally in the employees' interest to have generous retirement benefits. While this may be true, it is not necessary for the argument's conclusion. The argument only focuses on the budget shortfall and the need for cuts in retirement benefits to avoid job losses. Therefore, this answer choice is not required for the argument.

(D) Cutting the retirement benefits would help resolve the agency's budget shortfall.

  • This answer choice states that cutting retirement benefits would be an effective way to resolve the budget shortfall. If this were not true, then the argument's conclusion would fall apart. Therefore, this answer choice is a necessary assumption for the argument.

(E) The transportation employees' union will not accept cuts in retirement benefits if doing so will not allow more transportation employees to keep their jobs.

  • This answer choice introduces a condition that the union will not accept cuts in retirement benefits if it doesn't result in more employees keeping their jobs. However, the argument doesn't rely on this condition. It focuses on the budget shortfall and the need for cuts in retirement benefits to avoid job losses. Therefore, this answer choice is not required for the argument.

In conclusion, the correct assumption required for the argument is (D) Cutting the retirement benefits would help resolve the agency's budget shortfall.

1 Crore+ students have signed up on EduRev. Have you? Download the App
Test: Assumptions - Question 3

The tiny country of Minlandia does not produce its own television programming. Instead, the citizens of Minlandia, who generally are fluent not only in their native Minlandian, but also in Boltese, watch Boltese-language television programs from neighboring Bolta. Surveys show that the Minlandians spend on average more hours per week reading for pleasure and fewer hours per week watching television than people anywhere else in the world. A prominent psychologist accounts for the survey results by explaining that people generally prefer to be entertained in their native language even if they are perfectly fluent in other languages.

The explanation offered by the psychologist accounts for the Minlandian’s behavior only if which one of the following is assumed?

Detailed Solution for Test: Assumptions - Question 3

The psychologist's explanation is that people generally prefer to be entertained in their native language, even if they are fluent in other languages. The survey results show that Minlandians spend more time reading for pleasure and less time watching television. The correct answer choice will provide an assumption that supports this explanation.

(A) Some Minlandians derive no pleasure from watching television in a language other than their native Minlandian.

  • This answer choice suggests that some Minlandians may not enjoy watching television in a language other than their native language. While this could be a possible explanation, it does not directly support the psychologist's explanation that people generally prefer entertainment in their native language.

(B) The study of Boltese is required of Minlandian children as part of their schooling.

  • This answer choice suggests that all Minlandian children are required to study Boltese as part of their schooling. However, it does not provide a direct connection to the survey results or the psychologist's explanation. It is also not necessary for the Minlandians to study Boltese in school for the psychologist's explanation to hold true.

(C) The proportion of bilingual residents to the total population is greater in Minlandia than anywhere else in the world.

  • This answer choice suggests that Minlandia has a higher proportion of bilingual residents compared to other countries. While this could be a contributing factor to the survey results, it does not directly support the psychologist's explanation that people generally prefer entertainment in their native language.

(D) At least some of what the Minlandians read for pleasure is in the Minlandian language.

  • This answer choice directly supports the psychologist's explanation. If at least some of what Minlandians read for pleasure is in the Minlandian language, it suggests that they have a preference for entertainment in their native language. This aligns with the survey results showing that Minlandians spend more time reading for pleasure and less time watching television.

(E) When Minlandians watch Boltese television programs, they tend to ignore the fact that they are hearing a foreign language spoken.

  • This answer choice suggests that Minlandians tend to ignore the fact that Boltese is a foreign language when they watch television programs. While this could be a possibility, it does not directly support the psychologist's explanation that people generally prefer entertainment in their native language.

In conclusion, the correct answer is (D) because it directly supports the psychologist's explanation by suggesting that at least some of what Minlandians read for pleasure is in their native language, indicating a preference for entertainment in their native language.

Test: Assumptions - Question 4

In the past century formerly consensual perceptions of the human palate have changed significantly. Man's culinary likes and dislikes have proved to be less culturally ingrained in society than was once thought and dishes foreign to many cultures have proliferated in these cultures across the globe in the last 100 years. A company marketing packaged goods from Country X has deduced that it will be profitable to market its packaged goods of Country X's cuisine to Country Y.

The company's deduction must rely on all the following assumptions EXCEPT:

Detailed Solution for Test: Assumptions - Question 4

The question is asking us to identify the assumption that the company's deduction does NOT rely on. In other words, we need to find the assumption that is not necessary for the company's conclusion to be valid.

(A) Country Y will accept packaged goods.

  • This assumption is necessary for the company's deduction to be valid. If Country Y does not accept packaged goods, then the company's plan to market its packaged goods will not be successful.

(B) Country Y will enjoy the specific foods of Country X.

  • This assumption is necessary for the company's deduction to be valid. If the people in Country Y do not enjoy the specific foods of Country X, then the company's packaged goods will not be popular or profitable.

(C) There is a way to transfer packaged goods from Country X to Country Y.

  • This assumption is necessary for the company's deduction to be valid. If there is no way to transfer the packaged goods from Country X to Country Y, then the company will not be able to sell its products in Country Y.

(D) The company is the first from Country X to export packaged goods to Country Y.

  • This is the correct answer choice. The company's deduction does not rely on being the first to export packaged goods to Country Y. Even if other companies from Country X have already exported packaged goods to Country Y, it does not necessarily invalidate the profitability of the company's plan.

(E) The price the citizens of Country Y are willing to pay for the company's packaged goods exceeds the price of research, production, and shipping the company must invest to sell its goods in Country Y.

  • This assumption is necessary for the company's deduction to be valid. If the price citizens of Country Y are willing to pay does not cover the costs of research, production, and shipping, then the company's plan will not be profitable.

Therefore, the correct answer is (D) The company is the first from Country X to export packaged goods to Country Y.

Test: Assumptions - Question 5

While e-reading is more convenient by far, it has drawbacks: you can have books purloined by publishers and vendors over legal squabbles, the battery can die, and you need to turn your e-reader off for airplane takeoffs and landings. Regardless of how pervasive technology becomes in our literary lives, people will forever long for the tactile experience of reading a tangible book - bought from a tangible store. Therefore, online stores and e-books do not pose a threat to independent bookstores or larger chains.

The argument above assumes the validity of which of the following premises?

Detailed Solution for Test: Assumptions - Question 5

The argument states that despite the convenience of e-reading, people will always long for the tactile experience of reading a tangible book and that online stores and e-books do not pose a threat to independent bookstores or larger chains.

(A) No larger book store chain will go out of business in the foreseeable future.

  • This assumption is not necessary for the argument's conclusion to hold. The argument is focused on the preference for physical books over e-books and online stores, rather than the financial viability of larger book store chains.

(B) Independent bookstores should become better positioned to compete with larger chains and online stores.

  • This assumption is not necessary for the argument's conclusion. The argument is based on the assumption that people will always long for the tactile experience of reading a tangible book, regardless of the competitiveness of independent bookstores.

(C) A representative consumer factors in the tactile sensation of a book in the decision process to purchase that book.

  • This assumption is necessary for the argument's conclusion to hold. The argument relies on the assumption that the tactile sensation of a book is an important factor in the decision process to purchase a book.

(D) Online stores and e-books will continue to grow in popularity.

  • This assumption is not necessary for the argument's conclusion. Even if online stores and e-books do not continue to grow in popularity, the argument still maintains that people will prefer physical books.

(E) The subject matter of a book is the deciding factor in the decision to own that book.

  • This assumption is not necessary for the argument's conclusion. The argument is focused on the preference for the tactile experience of a physical book rather than the deciding factor of the subject matter.

Therefore, the correct answer is (C) A representative consumer factors in the tactile sensation of a book in the decision process to purchase that book.

Test: Assumptions - Question 6

A recent study analyzing the effects of day care on the productivity of the employees at a machine-parts company showed astonishing results. Forty percent of those factories that had day care programs produced more than 20,000 machine parts annually, while only 10 percent of those factories that did not have day care programs produced over 20,000 machine parts annually. This shows that day care has a positive effect on a company's productivity.

The argument above relies on which of the following assumptions?

Detailed Solution for Test: Assumptions - Question 6

The argument states that 40 percent of the factories with day care programs produce more than 20,000 machine parts annually, while only 10 percent of the factories without day care programs achieve the same level of production. The conclusion drawn from this is that day care has a positive effect on a company's productivity.

To identify the assumption, we need to find the statement that must be true in order for the conclusion to be valid. Let's go through the answer choices:

(a) At least 30 percent of those factories that do not have day care programs would now be producing over 20,000 machine parts annually if those factories had implemented day care programs.
This assumption is not necessary for the conclusion to hold true. The argument does not state that any specific percentage of factories would achieve higher production with day care programs.

(b) The factories with day care programs and the factories without day care programs are similar in ways that affect the ability to produce a high volume of machine parts annually.
This assumption is essential for the conclusion to be valid. The argument compares the productivity of factories with and without day care programs. If these factories were not similar in other factors that affect productivity, then the difference in output could be attributed to other variables rather than the presence or absence of day care programs.

(c) The factories that did not implement day care programs did so because their production level was so low that they could not afford to start new programs.
This assumption is not necessary for the conclusion. The argument does not provide any information about why factories without day care programs chose not to implement them.

(d) The factories with day care programs are better designed and more efficiently run than the factories without day care programs.
This assumption is not necessary for the conclusion. The argument only discusses the productivity levels of the factories with and without day care programs, not their design or efficiency.

(e) The factories without day care programs produce machine parts of a significantly higher quality than do the factories with day care programs.
This assumption is not necessary for the conclusion. The argument does not mention anything about the quality of the machine parts produced by the factories.

In conclusion, option (b) is the correct assumption because it ensures that the comparison between factories with and without day care programs is valid by assuming that they are similar in other factors that affect productivity.

Test: Assumptions - Question 7

The restrictions on what one can carry inside an airplane are surely too severe. Airlines these days issue a long checklist of items that cannot be carried inside an aircraft making it cumbersome for the travelers to keep a track of all these items. In any case, barely two percent of travelers are ever caught with these restricted items, so it’s best if these restrictions are removed.

Which of the following is an assumption on which the argument depends?

Detailed Solution for Test: Assumptions - Question 7

Let's go through each answer choice and evaluate its relevance to the argument and its potential as an assumption.

(A) The restrictions themselves do not discourage people from carrying restricted items on to airplanes.

  • This assumption is relevant to the argument because if the restrictions are effective in discouraging people from carrying restricted items, then the argument's claim that the restrictions are unnecessary would be weakened. Therefore, the argument depends on assuming that the restrictions do not effectively deter people from carrying prohibited items. This makes answer choice (A) a necessary assumption for the argument.

(B) The restrictions have been implemented all across the world.

  • The argument does not explicitly discuss the global implementation of these restrictions. It focuses on the inconvenience caused to travelers and the low rate of detection of restricted items. The global implementation of restrictions is not necessary for the argument's logic, so answer choice (B) is not directly relevant.

(C) There is no scope for corrupt practices on the part of those enforcing these restrictions.

  • This assumption is not directly related to the argument. The argument is focused on the inconvenience caused to travelers and the low rate of detection, not on corruption among those enforcing the restrictions. Therefore, answer choice (C) is not a necessary assumption for the argument.

(D) There have been instances when an innocent traveler has been wrongly detained by over cautious airline personnel.

  • While this assumption introduces the possibility of innocent travelers being wrongly detained, it does not directly impact the argument's main claim that the restrictions should be removed. It introduces a different issue related to the potential mistakes of airline personnel. Therefore, answer choice (D) is not a necessary assumption for the argument.

(E) Criminals are hardly likely to carry restricted items with them on an airline.

  • This assumption is not directly relevant to the argument. The argument focuses on the inconvenience caused to travelers and the low rate of detection, not on the likelihood of criminals carrying restricted items. Therefore, answer choice (E) is not a necessary assumption for the argument.

In summary, the assumption that the argument depends on is (A) The restrictions themselves do not discourage people from carrying restricted items on to airplanes.

Test: Assumptions - Question 8

Although the charter of Westside School states that the student body must include some students with special educational needs, no students with learning disabilities have yet enrolled in the school. Therefore, the school is currently in violation of its charter.

The conclusion of the argument follows logically if which one of the following is assumed?

Detailed Solution for Test: Assumptions - Question 8

Let's analyze each answer choice and determine which one, when assumed, would lead to the conclusion that the school is currently in violation of its charter.

(A) All students with learning disabilities have special educational needs.
This answer choice is not necessary to reach the conclusion because even if there are students with learning disabilities who don't have special educational needs, it wouldn't change the fact that the school has violated its charter.

(B) The school currently has no students with learning disabilities.
This answer choice supports the conclusion that the school is in violation of its charter. If the charter requires the inclusion of students with learning disabilities and none are currently enrolled, then the school is not fulfilling its obligations.

(C) The school should enroll students with special educational needs.
This answer choice is not necessary to reach the conclusion. It may be a reasonable assumption, but the conclusion does not depend on it.

(D) The only students with special educational needs are students with learning disabilities.
This answer choice is not necessary to reach the conclusion. The charter of the school might include other students with special educational needs aside from those with learning disabilities.

(E) The school's charter cannot be modified to avoid its being violated.
This answer choice is the correct assumption that leads to the conclusion. If the charter cannot be modified and the requirement to include students with special educational needs, including those with learning disabilities, remains unfulfilled, then the school is indeed in violation of its charter.

Therefore, the correct answer is (E).

Test: Assumptions - Question 9

Columnist: A democratic society cannot exist unless its citizens have established strong bonds of mutual trust. Such bonds are formed and strengthened only by participation in civic organizations, political parties, and other groups outside the family. It is obvious then that widespread reliance on movies and electronic media for entertainment has an inherently corrosive effect on democracy.

Which one of the following is an assumption on which the columnist’s argument depends?

Detailed Solution for Test: Assumptions - Question 9

The columnist argues that a democratic society requires strong bonds of mutual trust among its citizens, which are formed and strengthened through participation in civic organizations, political parties, and other groups outside the family. The columnist then claims that widespread reliance on movies and electronic media for entertainment has a corrosive effect on democracy.

To support this argument, the columnist must rely on certain assumptions. Let's examine each answer choice:

(A) Anyone who relies on movies and electronic media for entertainment is unable to form a strong bond of mutual trust with a citizen.

  • This assumption is too extreme and unsupported by the argument. The columnist does not claim that relying on movies and electronic media makes it impossible to form strong bonds of mutual trust.

(B) Civic organizations cannot usefully advance their goals by using electronic media.

  • This assumption is not necessary for the columnist's argument. The argument focuses on the impact of movies and electronic media on democracy as a whole, rather than the effectiveness of civic organizations in achieving their goals.

(C) Newspapers and other forms of print media strengthen, rather than weaken, democratic institutions.

  • This assumption is not relevant to the argument made by the columnist. The argument specifically discusses movies and electronic media, not print media.

(D) Relying on movies and electronic media for entertainment generally makes people less likely to participate in groups outside their families.

  • This assumption aligns with the columnist's argument. The claim that widespread reliance on movies and electronic media has a corrosive effect on democracy implies that it reduces people's participation in groups outside their families. This assumption supports the overall argument.

(E) People who rely on movies and electronic media for entertainment are generally closer to their families than are those who do not.

  • This assumption is not relevant to the argument. The argument focuses on the impact of movies and electronic media on democracy, not the relationship between individuals and their families.

Therefore, the correct answer is (D) - relying on movies and electronic media for entertainment generally makes people less likely to participate in groups outside their families. This assumption supports the argument that widespread reliance on such media has a corrosive effect on democracy.

Test: Assumptions - Question 10

An acre of average farmland produces only about 400 pounds of grain amaranth, as against 2,400 pounds per acre, or six times as much, for wheat. It follows that whenever the grain-amaranth price is projected to be more than six times the projected price of wheat, farmers wishing to maximize profits will grow grain amaranth rather than wheat.

The argument above is based on which of the following assumptions?

Detailed Solution for Test: Assumptions - Question 10

The argument states that if the projected price of grain amaranth is more than six times the projected price of wheat, farmers will choose to grow grain amaranth to maximize profits. The conclusion is based on an implicit assumption. We need to identify the assumption that, if true, strengthens the conclusion.

Let's evaluate each answer choice:

(A) An acre's worth of grain amaranth is no more expensive to grow and bring to market than an acre's worth of wheat.

  • This assumption is necessary for the conclusion to hold. If grain amaranth were more expensive to grow and bring to market than wheat, the potential profit from growing grain amaranth might not exceed that of wheat, even if the price ratio is favorable.

(B) There is no crop that produces a higher yield in terms of pounds harvested per acre than wheat.

  • This assumption is not necessary for the conclusion. Even if other crops yield more pounds per acre than wheat, it does not necessarily mean that the profit from growing those crops would be higher than growing grain amaranth.

(C) By choosing which crops to grow, farmers can exert a significant influence on the prices of those crops.

  • This assumption is not necessary for the conclusion. Even if farmers cannot influence the prices of crops, the conclusion can still hold true if the price ratio mentioned in the argument exists.

(D) Farmers are no less motivated by the desire to maximize profits than are other occupational groups.

  • This assumption is not necessary for the conclusion. The motivation of farmers relative to other occupational groups is irrelevant to the argument.

(E) Prices of grain crops can change faster than farmers can change the acreage devoted to various grain crops.

  • This assumption is not necessary for the conclusion. The argument is based on projected prices, not immediate changes in prices.

Therefore, the correct answer is (A) An acre's worth of grain amaranth is no more expensive to grow and bring to market than an acre's worth of wheat. This assumption ensures that the potential profit from growing grain amaranth exceeds that of wheat when the price ratio condition is met.

18 docs|139 tests
Information about Test: Assumptions Page
In this test you can find the Exam questions for Test: Assumptions solved & explained in the simplest way possible. Besides giving Questions and answers for Test: Assumptions, EduRev gives you an ample number of Online tests for practice

Top Courses for GMAT

Download as PDF

Top Courses for GMAT